문제지 PDF 파일을 로드하고 있습니다. 한참 걸릴 수도 있어요 ㅠㅠ 30초 이상 걸리면 새로고침을 한번 해보세요.

2013 7급 국가직 통신이론

1번

  1. a>0a\gt 0일 때

    f(at)ejωtdt=f(t~)ejωat~1adt~=1aF(ωa)\begin{equation} \begin{split} \int_{-\infty}^{\infty}f(at)e^{-j\omega t}dt &=\int_{-\infty}^{\infty}f(\tilde{t})e^{-j\frac{\omega}{a}\tilde{t}}\frac{1}{a}d\tilde{t}\\ &=\frac{1}{a}F\left(\frac{\omega}{a} \right) \end{split} \end{equation}

    이다. a<0a\lt 0일 때는 직접 해보자.

  2. df(t)dtejωtdt=f(t)ejωtf(t)(jω)ejωtdt\int_{-\infty}^{\infty} \frac{df(t)}{dt}e^{-j\omega t}dt=\left.f(t)e^{-j\omega t}\right|_{-\infty}^{\infty}-\int_{-\infty}^{\infty}f(t)(-j\omega)e^{-j\omega t}dt

    인데, 푸리에 변환이 수렴하기 위해서는 f()=f()=0f(\infty)=f(-\infty)=0이어야 한다. 따라서 위 적분은

    df(t)dtejωtdt=f(t)(jω)ejωtdt=(jω)f(t)ejωtdt=(jω)F(ω)\begin{equation} \begin{split} \int_{-\infty}^{\infty} \frac{df(t)}{dt}e^{-j\omega t}dt&=\int_{-\infty}^{\infty}f(t)(j\omega)e^{-j\omega t}dt\\ &=(j\omega)\int_{-\infty}^{\infty}f(t)e^{-j\omega t}dt\\ &=(j\omega)F(\omega) \end{split} \end{equation}

    이다. 미분을 더 하면 결국 (jω)(j\omega)가 미분회수만큼 곱해질 것이므로

    dnf(t)dt(jω)nF(ω)\frac{d^nf(t)}{dt}\leftrightarrow (j\omega)^nF(\omega)

    이다.

  3. F(t)ejωtdt=f(t~)ejtt~dt~ejωtdt=f(t~)ej(ω+t~)tdtdt~=f(t~)2πδ(ω+t~)dt~=2πf(ω)\begin{equation} \begin{split} \int_{-\infty}^{\infty}F(t)e^{-j\omega t}dt&= \int_{-\infty}^{\infty} \int_{-\infty}^{\infty} f(\tilde{t})e^{-jt\tilde{t}}d\tilde{t}e^{-j\omega t}dt\\ &= \int_{-\infty}^{\infty} f(\tilde{t})\int_{-\infty}^{\infty} e^{-j(\omega+\tilde{t})t}dtd\tilde{t}\\ &= \int_{-\infty}^{\infty}f(\tilde{t})\cdot 2\pi \delta(\omega+\tilde{t})d\tilde{t}\\ &= 2\pi f(-\omega) \end{split} \end{equation}

    이다. 여기서

    δ(f)=ej2πftdtδ(ω)=δ(2πf)=ej2π2πftdt=ej2πft~12πdt~=12πejωt~dt~ejωtdt=2πδ(ω)\begin{equation} \begin{split} \delta(f)&=\int_{-\infty}^{\infty}e^{j2\pi ft}dt\\ \Rightarrow \delta(\omega)&=\delta(2\pi f)\\ &=\int_{-\infty}^{\infty}e^{j2\pi 2\pi ft}dt\\ &=\int_{-\infty}^{\infty}e^{j2\pi f\tilde{t}}\frac{1}{2\pi}d\tilde{t}\\ &=\frac{1}{2\pi}\int_{-\infty}^{\infty}e^{j\omega \tilde{t}}d\tilde{t}\\ \Rightarrow \int_{-\infty}^{\infty}e^{j\omega t}dt&=2\pi \delta(\omega) \end{split} \end{equation}

    임을 이용하였다.(지수의 부호는 -가 되어도 마찬가지이다. 직접 해보라.)

  4. f(tτ)ejωtdt=f(t~)ejω(t~+τ)dt~=f(t~)ejωt~dt~ejωτ=F(ω)ejωτ\begin{equation} \begin{split} \int_{-\infty}^{\infty}f(t-\tau)e^{-j\omega t}dt&=\int_{-\infty}^{\infty} f(\tilde{t})e^{-j\omega(\tilde{t}+\tau)}d\tilde{t}\\ &=\int_{-\infty}^{\infty} f(\tilde{t})e^{-j\omega \tilde{t}}d\tilde{t}e^{-j\omega\tau}\\ &=F(\omega)e^{-j\omega \tau} \end{split} \end{equation}

    이다. 시간이 늦어지면 위상도 마찬가지로 늦어진다.

따라서 답은 4번 이다.

2번

F{x(t)}=X(f)=rect(f)={1f<0.50f>0.5\begin{equation} \begin{split} \mathcal{F}\{x(t)\}&=X(f)\\ &=rect(f)\\ &=\begin{cases} 1&f\lt 0.5\\ 0&f\gt 0.5 \end{cases} \end{split} \end{equation}

이다. 따라서 X(f)2=X(f)|X(f)|^2=X(f)임과 Parseval’s Theorem을 이용하면 x(t)x(t)의 총 에너지는

Ex=X(f)2df=0.50.5df=1\begin{equation} \begin{split} E_x&=\int_{-\infty}^{\infty}|X(f)|^2df\\ &=\int_{-0.5}^{0.5}df\\ &=1 \end{split} \end{equation}

이다. 따라서 답은 2번 이다.

3번

채널 대여곡이 10 MHz이다. 따라서 기저대역의 메시지 신호의 최대 대역폭은 5 MHz이고, 이의 2배인 10 MHz의 속도로 심볼을 보낼 수 있다. 한편 8-PSK이므로 심볼당 비트는 log28=3log_2 8=3 비트이므로 최대 전송 속도는

3×10=30 Mbps3\times 10=30\text{ Mbps}

이니 답은 4번 이다.

4번

C=Wlog2(1+SNR)=10×log2(1+70×10310×103)=10×log28=30 Mbps\begin{equation} \begin{split} C&=Wlog_2{(1+SNR)}\\ &=10\times log_2{\left(1+\frac{70\times 10^{-3}}{10\times 10^{-3}}\right)}\\ &=10\times log_2 8\\ &=30\text{ Mbps} \end{split} \end{equation}

이므로 답은 3번 이다.

5번

12X\frac{1}{2}X의 분산은 계수의 제곱이 원래 분산에 곱해진 것과 같으므로 14\frac{1}{4}이다. 한편 독립인 확률변수들의 합의 분산은 각각의 분산의 합과 같으므로

σN=14+1=1.25\begin{equation} \begin{split} \sigma_N&=\frac{1}{4}+1\\ &=1.25 \end{split} \end{equation}

이다. 따라서 답은 3번 이다.

6번

  1. https://ham.stackexchange.com/questions/21400/is-fm-effectively-spread-spectrum 에 따르면 FM은 결론적으로 확산대역 통신이 아니다. 가장 추천을 많이 받은 답변에서는 확산대역 통신은 원하는 신호와 상관이 없는 간섭의 영향을 낮추고 원하는 신호를 강하게 만들어서 추출할 수 있는 반면, FM은 그냥 강한 신호를 추출해버리는 차이가 있다. 또한 두 번째 답변에 따르면 FM은 그 자체로 변조 방식이고, 확산대역 통신은 변조된 신호를 다시 넓게 확산시키는 차이가 있다. 이러한 이유로 FM은 확산대역 통신이 아니다.
  2. 그렇다. 처프 방식도 있다고 한다.
  3. 그렇다. 칩 전송률이 커지면 전송 대역폭도 넓어진다.
  4. 그렇다. 대역을 더 넓게 확산시킬수록 간섭전력은 더 퍼지기 때문이다.

따라서 답은 1번 이다.

7번

수신된 심볼을 rr, 송신된 심볼을 tt라 하자. 구하고자 하는 것은

P(r=0)=P(t=0)P(r=0t=0)+P(t=1)P(r=0t=1)=0.4×0.9+0.6×0.1=0.42\begin{equation} \begin{split} P(r=0)&=P(t=0)P(r=0|t=0)+P(t=1)P(r=0|t=1)\\ &=0.4\times 0.9+0.6\times 0.1\\ &=0.42 \end{split} \end{equation}

이므로 답은 2번 이다.

8번

m(t)m(t)의 최고 주파수는 k=5k=5일 때의 주파수, 즉 5 Hz이다. 따라서 메시지 신호의 대역폭은 5 Hz이다. 이를 DSB-FC 변조하면 음의 주파수 영역도 반송파 주파수의 왼쪽에 나타나는 양의 주파수가 되므로 변조된 신호의 대역폭은 메시지 신호의 대역폭의 2배인 10 Hz가 된다. 따라서 답은 2번 이다.

9번

심볼 지속 시간이 2초이므로 2개씩 잘라놓으면 그 안에 2비트가 들어있다. 코사인을 x축에 놓고, 이를 반시계 방향으로 9090^\circ 회전한(=한번 미분한) sin-sin을 y축에 놓아보자.

  1. 첫 2초는 sin-sin 형태이므로 (x축으로부터 시계 방향으로 4545^\circ 회전한 성분)×(1)\times(-1)sin=ysin=-y축에서 4545^\circ 회전한 성분에 -를 곱한 성분이 더해져야 한다. 즉 dI=1,dQ=0d_I=1,d_Q=0이다.
  2. 두 번째 2초는 sinsin 형태이므로 (x축으로부터 반시계 방향으로 4545^\circ 회전한 성분)×(1)\times(-1)sinsin으로부터 4545^\circ회전한 성분이 더해져야 한다. 즉 dI=0,dQ=1d_I=0, d_Q=1이다.
  3. 세 번째 2초는 coscos 형태이므로 x축으로부터 반시계 방향으로 4545^\circ 회전한 성분과 sinsin으로부터 4545^\circ회전한 성분이 더해져야 한다. 즉 dI=dQ=1d_I=d_Q=1이다.
  4. 네 번째 2초는 cos-cos 형태이므로 3번의 반대이다. 즉 dI=dQ=0d_I=d_Q=0이다.

이로부터 답은 1번 임을 알 수 있다.

10번

  1. ㄱ: 그렇다고 한다. 왜 그런지는 http://atlantarf.com/FSK_Modulation.php를 참고하라.
  2. ㄴ: 그렇다. 두 비트 모두 바뀔 때이다.
  3. ㄷ: 그렇다. 비트를 사각형 함수 대신 대신 정현파로 표현한 형태이다.
  4. ㄹ: 그렇다. I채널과 Q채널은 독립이기 때문이다.

따라서 답은 4번 이다.

11번

  1. 그렇다. WSS이므로 t~=tτ\tilde{t}=t-\tau를 대입해도 같은 값을 가짐을 이용하면
    E[X(t~)X(t~+τ)]=E[X(tτ)X(t)]=E[X(t)X(t+τ)]\begin{equation} \begin{split} E[X(\tilde{t})X(\tilde{t}+\tau)]&=E[X(t-\tau)X(t)]\\ &=E[X(t)X(t+\tau)] \end{split} \end{equation}
    이므로 우함수이다.
  2. 그렇다. 아무리 상관성이 강해봐야 동시(τ=0)(\tau=0)일 때보다 강하진 못할 것이다.
  3. 그렇지 않다. 평균 전력과 같다. RX(0)=E[X2(t)]R_X(0)=E[X^2(t)]이기 때문이다.
  4. 그렇다. 이를 위너-킨친 정리라고 한다.

따라서 답은 3번 이다.

12번

미분기는 주파수 변화에 비례하는 진폭을 만들어낸다. 따라서 답은 1번 이다.

13번

  1. 그렇다.
  2. 10000/10=100010000/10=1000이므로 그렇다.
  3. CDMA는 직접확산을 사용하므로 전송 대역폭은 정보신호의 대역폭보다 훨씬 크다.
  4. 그렇다. 그래서 상관을 구하면 내가 원하는 신호의 전력은 크게 나오지만 다른 신호들은 낮은 전력을 갖게 되어 무시가 가능하다. 다만 다른 신호들의 전력이 너무 강하다면 이 신호들을 무시할 수 없게 된다. 따라서 각 단말기들의 송신신호를 기지국이 수신했을 떄의 전력을 같게 해야 한다. 그래야 기준을 세워서 구분이 가능하기 때문이다.

따라서 답은 3번 이다.

14번

  1. 그렇다.
  2. 최소 거리의 절반보다 적은 비트들의 오류를 정정할 수 있다. 즉 최대 3개 비트의 오류를 정정할 수 있다. 그렇게 되는 이유는, 심볼들로 이루어진 공간에서 최소 거리의 절반 안에 수신 심볼이 찍히면 가장 가까운 심볼을 골랐을 때 원래 송신된 심볼로 무사히 선택되지만, 최소 거리의 절반을 넘어가버리면 다른 심볼이 더 가깝게 되어 잘못 복원되기 때문이다.
  3. 그렇다.
  4. 최소 거리보다 1만큼 작은 수의 오류 비트 수를 검출할 수 있다. 최소 거리만큼 떨어져버리면 그냥 다른 심볼이기 때문이다.

따라서 답은 2번 이다.

15번

  1. fIF=fLOfcf_IF=|f_{LO}-f_c|

    이고 정해져 있지 가변적인 값을 갖진 않는다. 가변적이라면 중간주파수에 따라서 회로의 작동 대역이 전부 바뀌어야 하는데, 그럴 바에야 중간주파수 없이 하는 편이 나을 것이다.

  2. 그렇다. fLO+fIFf_{LO}+f_{IF}의 주파수도 수신되어버리기 때문이다.

  3. 그렇다. 가청 주파수는 보통 20 kHz인 반면, AM 라디오의 경우 fIFf_{IF}는 455 kHz 정도의 값을 갖는다. 중간 주파수가 높아야 회로가 요구하는 Q팩터 조건이 완화된다.

  4. 그렇다.

따라서 답은 1번 이다.

16번

  1. 그렇다.
  2. 채널 상태에 따라 안좋은 부반송파 대신 더 상태가 좋은(상관대역 안에 들어가는) 부반송파를 할당받을 수 있다.
  3. 그렇다. 부반송파들의 위상이 맞을 때 높은 진폭이 만들어지기 때문이다.
  4. 그렇다.

따라서 답은 2번 이다.

17번

  1. 그렇다. 서로 다른 부호에 대해 상관계수가 낮은 부호를 사용한다.
  2. 잘 모르겠으니 일단 넘어가보자.
  3. 그렇지 않다. 기지국이 수신하는 전력이 일정하도록 송신 전력을 조절한다. 그래야 수신 전력에 대한 기준을 정해서 원하는 신호인지 아닌지 구분할 수 있다.
  4. 2개의 기지국과 동시에 채널을 만들어서 통신하면서 더 좋은 쪽을 취하는 식으로 경로 다이버시티를 이용한다고 한다.

따라서 답은 3번 이다.

18번

3 kHz로 제한된 음성 신호들이므로 나이퀴스트율을 만족하기 위해서는 각각 6 kHz의 대역폭이 필요하다. 이러한 게 12개 있으므로 우선 72 kHz가 필요하다. 그리고 채널 사이에 1 kHz가 필요한데, DSB로 변조하였으니 24개 채널이 있는 것과 같다. 따라서 23 kHz의 간격들이 필요하다. 그러므로

6×12+11×2×1+1×1=95 kHz6\times12+11\times2\times1+1\times1=95 \text{ kHz}

의 대역폭이 필요하므로 답은 2번 이다.

19번

채널부호화 전 전송률이 2 Mbps인데 부호화율이 35\frac{3}{5}이므로 필요한 전송률은 2×53=103>32\times\frac{5}{3}=\frac{10}{3}\gt3이다. 따라서 최소 4비트가 심볼에 실려야 하는데 이를 만족하는 보기는 4번 뿐이다.

20번

잡음의 전력 스펙트럼 밀도가

SN(f)=N02S_N(f)=\frac{N_0}{2}

이다. 따라서 잡음 전력은

N02×2×2W=2N0W\frac{N_0}{2}\times 2\times 2W=2N_0W

이므로 답은 3번 이다.